Understanding Convolution Integrals: Explained and Examples | Homework Help

Click For Summary
The discussion centers on the challenges of understanding convolution integrals in relation to a system's impulse response and various input functions. The user provides an example with an impulse response g(t) and a step function, successfully deriving the output. However, confusion arises when analyzing a piecewise input function, leading to questions about the limits of integration and the relevance of certain terms in the convolution process. The user expresses frustration over the necessity of considering parts of the input that seem irrelevant to the output for specific time intervals. Ultimately, the discussion emphasizes the importance of treating each segment of the input independently while recognizing that the system's response is influenced by past inputs.
dfx
Messages
59
Reaction score
1

Homework Statement



Right I'm having a lot of problems with convolution in general. I'll give an example of a question that I understand and why I think I understand it... and then one I don't at all.

So:

Consider a system with the impulse response g(t) = 0 for t<0, e^{-5t} for t \geq 0.

Find the output for input f(t) = H(t) (step function).

So y(t) = \int g(t-\tau)f(\tau)d\tau between t and -\infty

= \int e^{-5(t-\tau)}H(\tau)d\tau between t and -\infty

= \int e^{-5(t-\tau)}.1.d\tau between t and 0. This is because the step function takes a value of 1 for t >= 0 hence the limits change to 0 and t?


Now for the same system say you have an input of:

f(t) = (0, t<0) ... (a) ; (v, 0<t<k) ... (b); (0, t>k) ... (c)

To find the output you need to perform 3 integrals: 1 for (a), 1 for (b) and 1 for (c).

According to my course notes these 3 integrals are:

1. part (a):

y(t) = \int g(t-\tau)0d\tau between t and -\infty

This sort of makes sense but why isn't the upper limit 0 as surely (a) is only for (t<0).

2. part (b):
y(t) = \int g(t-\tau)0d\tau between 0 and -\infty
+ \int g(t-\tau)vd\tau between t and 0.

I don't understand this bit. Firstly, why the initial bit between 0 and -infty ... surely it's unnecessary as the amplitude is 0 as in the previous example for the step input we didn't bother with t<0. Secondly for the second bit with amplitude v why on Earth are the limits t and 0. Surely they should be k and 0 as the amplitude is only v between k and 0?

3. part (c):

y(t) = \int g(t-\tau)0d\tau between 0 and -\infty
+ \int g(t-\tau)vd\tau between k and 0.
+ \int g(t-\tau).0.d\tau between t and k.

How are the first 2 lines in this integral even relevant to part (c) which is only for t>k where the amplitude is 0 (ie the way I see it only the third line is relevant).

Sorry for the rather unconventional post but this has been confusing me for a few months and having spent hours on it I just can't seem to understand what on Earth is going on! Any help/feedback much appreciated. Cheers.
 
Physics news on Phys.org
I think I understand now. For future reference, treat each each bit individually and remember a system in time remembers the past but doesn't know the future! Each separate bit is the sum of the previous responses, except the system knows what's happened before... if that makes sense.
 
Question: A clock's minute hand has length 4 and its hour hand has length 3. What is the distance between the tips at the moment when it is increasing most rapidly?(Putnam Exam Question) Answer: Making assumption that both the hands moves at constant angular velocities, the answer is ## \sqrt{7} .## But don't you think this assumption is somewhat doubtful and wrong?

Similar threads

Replies
2
Views
2K
  • · Replies 1 ·
Replies
1
Views
1K
  • · Replies 1 ·
Replies
1
Views
2K
  • · Replies 2 ·
Replies
2
Views
2K
  • · Replies 14 ·
Replies
14
Views
2K
  • · Replies 23 ·
Replies
23
Views
3K
  • · Replies 4 ·
Replies
4
Views
2K
  • · Replies 11 ·
Replies
11
Views
7K
Replies
6
Views
2K
  • · Replies 11 ·
Replies
11
Views
2K